Last visit was: 28 Apr 2024, 03:20 It is currently 28 Apr 2024, 03:20

Close
GMAT Club Daily Prep
Thank you for using the timer - this advanced tool can estimate your performance and suggest more practice questions. We have subscribed you to Daily Prep Questions via email.

Customized
for You

we will pick new questions that match your level based on your Timer History

Track
Your Progress

every week, we’ll send you an estimated GMAT score based on your performance

Practice
Pays

we will pick new questions that match your level based on your Timer History
Not interested in getting valuable practice questions and articles delivered to your email? No problem, unsubscribe here.
Close
Request Expert Reply
Confirm Cancel
SORT BY:
Date
Tags:
Show Tags
Hide Tags
Director
Director
Joined: 23 Apr 2019
Status:PhD trained. Education research, management.
Posts: 806
Own Kudos [?]: 1808 [3]
Given Kudos: 203
Send PM
Senior Manager
Senior Manager
Joined: 21 Aug 2018
Posts: 308
Own Kudos [?]: 681 [1]
Given Kudos: 19
Location: India
Send PM
VP
VP
Joined: 11 Aug 2020
Posts: 1262
Own Kudos [?]: 201 [0]
Given Kudos: 332
Send PM
Intern
Intern
Joined: 07 Feb 2021
Posts: 10
Own Kudos [?]: 3 [1]
Given Kudos: 58
Location: India
Schools: ISB '23 (S)
GMAT 1: 710 Q50 V39
Send PM
Re: When a patient failed to respond to prescribed medication, the doctor [#permalink]
1
Kudos
Understand Option C talks about only one side of the coin.

There are two:
1. Beverage was the killer
2. Dosage was half than the required

Option D clearly tackles both these points.




CEdward wrote:
When a patient failed to respond to prescribed medication, the doctor hypothesized that the dosage was insufficient. The doctor first advised doubling the dosage, but the patient's symptoms remained. It was then learned that the patient regularly drank an herbal beverage that often inhibits the medication's effect. The doctor then advised the patient to resume the initial dosage and stop drinking the beverage. The patient complied, but still showed no change. Finally, the doctor advised the patient to double the dosage and not drink the beverage. The patient's symptoms disappeared. Hence, the doctor's initial hypothesis was correct.

Which one of the following most accurately describes the manner in which the doctor's second set of recommendations and the results of its application support the doctor's initial hypothesis?

(A) They establish that the doctor's concerns about the healthfulness of the beverage were well founded. X

(B) They make it less plausible that the beverage actually contributed to the ineffectiveness of the prescribed medication. X

(C) They give evidence that the beverage was responsible for the ineffectiveness of the prescribed medication. Correct

(D) They suggest that the beverage was not the only cause of the ineffectiveness of the prescribed dosage. X

(E) They rule out the possibility that the doctor had initially prescribed the wrong medication for the patient's ailments. X

Can someone explain why it's D? The second set of recommendations in question is : "Finally, the doctor advised the patient to double the dosage and not drink the beverage." The patient's symptoms then disappeared.

How does that rule out that the beverage was NOT THE ONLY CAUSE?

I think C is correct because the fact that the symptoms disappeared when the dose was doubled and the patient wasn't drinking the beverage lends support to the fact that the dosage was insufficient...This was a direct follow-up to the first recommendation wherein the patient was told to stop drinking the beverage, but have only 1 dose.

GMATNinja thoughts?
Intern
Intern
Joined: 04 Apr 2018
Posts: 21
Own Kudos [?]: 19 [0]
Given Kudos: 148
Location: India
Concentration: General Management, Operations
GMAT 1: 700 Q50 V34 (Online)
GPA: 3
WE:Operations (Hospitality and Tourism)
Send PM
Re: When a patient failed to respond to prescribed medication, the doctor [#permalink]
When a patient failed to respond to prescribed medication, the doctor hypothesized that the dosage was insufficient. The doctor first advised doubling the dosage, but the patient's symptoms remained. It was then learned that the patient regularly drank an herbal beverage that often inhibits the medication's effect. The doctor then advised the patient to resume the initial dosage and stop drinking the beverage. The patient complied, but still showed no change. Finally, the doctor advised the patient to double the dosage and not drink the beverage. The patient's symptoms disappeared. Hence, the doctor's initial hypothesis was correct.

Few things to note here:
1. The prescribed dosage was not sufficient. Double dose was fine to relieve the patient.
2. The beverage was not the only cause as stopping its consumption and resuming the previously prescribed dosage did not relieve the patient. Increasing the dosage might have stopped any other source of the symptoms causing inhibition of the medicine.

Which one of the following most accurately describes the manner in which the doctor's second set of recommendations and the results of its application support the doctor's initial hypothesis?

(A) They establish that the doctor's concerns about the healthfulness of the beverage were well founded- IRRELEVANT. We are not talking about the healthfulness of the beverage.

(B) They make it less plausible that the beverage actually contributed to the ineffectiveness of the prescribed medication- INCORRECT. It was plausible that beverage could have contributed to the ineffectiveness but the extent is not talked about and hence we can assume something else was there.

(C) They give evidence that the beverage was responsible for the ineffectiveness of the prescribed medication- INCORRECT. Beverage was not responsible completely.

(D) They suggest that the beverage was not the only cause of the ineffectiveness of the prescribed dosage- CORRECT. Exact match with pre- thinking.

(E) They rule out the possibility that the doctor had initially prescribed the wrong medication for the patient's ailments- INCORRECT. Argument does not talk about the medication being right or wrong. It is about the hypothesis.
VP
VP
Joined: 10 Jul 2019
Posts: 1392
Own Kudos [?]: 543 [0]
Given Kudos: 1656
Send PM
When a patient failed to respond to prescribed medication, the doctor [#permalink]
We are looking for an answer that explains how the second set of recommendations and its results strengthen the doc’s first hypothesis: the first prescribed dosage did not help the patient because it was too low.


(B) tells us that they support this first hypothesis by making it LESS plausible that the beverage contributed to the medication not working.

If anything, the sequence of events would tend to show that the beverage MAY have contributed to the medication not working.

When the patient took the first higher dose while consuming the beverage, there was no relief. However, when the beverage was stopped and the patient took the higher dose the second time, she had success.

Removing the variable (“the beverage”) and seeing the effect disappear (“medication NOT working at higher dose”), if anything, would make it more plausible that the beverage in some way contributed to the medicine not working.

The doc’s initial hypothesis is that the first dosage is not enough to be effective for the patient. When it is learned that a beverage may be interfering with the efficacy of the medication, the doctor starts his second set of recommendations:

First, the patient tries the lower dosage again without the beverage. The medicine is not effective.

Then, the patient tries the higher dosage again without the beverage. This time the medicine is effective.

Showing that the lower dosage still did not work even without the consumption of the beverage supports the doc’s first claim by showing that something other than the beverage must have caused this first dosage to not work. After all, if it were only the beverage preventing the lower dosage from working, then when the “beverage variable” was removed during the second set of recommendations, the lower dosage would have worked.

Since the lower dosage did not work even when the patient did not drink the beverage and the medicine only worked when the dose was raised, this supports the doc’s first claim by showing that the beverage was not the only cause for the lower dose not working ——-> thereby making it more likely that another cause was at play (such as the dose being too low).

By ruling out the possibility that the beverage was the only thing preventing the lower dose from working, the second set of recommendations and its results tend to support the doc’s first claim that the initial dose was too low.

Thus, (D) explains this best.

Posted from my mobile device
GMAT Club Bot
When a patient failed to respond to prescribed medication, the doctor [#permalink]
Moderators:
GMAT Club Verbal Expert
6923 posts
GMAT Club Verbal Expert
238 posts
CR Forum Moderator
832 posts

Powered by phpBB © phpBB Group | Emoji artwork provided by EmojiOne